Vous êtes sur la page 1sur 23

Reconocimiento robotica Puntos: 1 Los sensores de efecto Hall son usados cuando: Seleccione al menos una respuesta. a. b. c.

El ambiente sea electromagntico Se quiera evitar el contacto mecnico Existan imnes en el ambiente Bien!

Muy bien! d. Se necesite evitar la produccin de rebotes Correcto Puntos para este envo: 1/1. Question 2 Puntos: 1 De acuerdo al video, seleccione la palabra adecuada que completa la siguiente frase: Los tres pasos bsicos con los que trabaja un robot son: Toma de _____________, procesamiento y generacin del movimiento Seleccione una respuesta. a. b. c. Mandos Clculos Sensores Incorrecto.

d. Seales OK Incorrecto Puntos para este envo: 0/1. Question 3 Puntos: 1 Este tipo de preguntas consta de dos proposiciones, una Afirmacin y una Razn, unidas por la palabra PORQUE. El estudiante debe examinar la veracidad de cada proposicin y la relacin terica que las une. Para responder este tipo de preguntas el estudiante debe leer toda la pregunta y sealar la respuesta elegida de acuerdo a las opciones presentadas. De acuerdo con la tabla presentada en la lectura, un voltaje de entrada de 0.0100 produce un valor binario de salida "00000000" PORQUE dicho voltaje no supera 8 bits de informacin Seleccione una respuesta. a. la afirmacin es VERDADERA, pero la razn es una proposicin FALSA. OK b. la afirmacin y la razn son VERDADERAS y la Incorrecto, La razn es razn es una explicacin CORRECTA de la afirmacin. falsa porque el voltaje es analgico.

c. la afirmacin y la razn son VERDADERAS, pero la razn NO es una explicacin CORRECTA de la afirmacin. d. la afirmacin es FALSA, pero la razn es una proposicin VERDADERA. Incorrecto Puntos para este envo: 0/1. Question 4 Puntos: 1 De las siguientes afirmaciones, seleccione la que considera verdadera teniendo en cuenta el video observado. Seleccione una respuesta. a. Los ingenieros elctricos no trabajan en el mbito de la robtica b. En robtica, es fcil reemplazar el funcionamiento del oido humano c. En latinoamrica no se trabaja en robtica Muy bien, las imgenes del video muestran d. La robtica esta involucrada en la robots trabajando en la construccin de industria automotriz OK automviles. Correcto Puntos para este envo: 1/1. Question 5 Puntos: 1 Este tipo de preguntas consta de dos proposiciones, una Afirmacin y una Razn, unidas por la palabra PORQUE. El estudiante debe examinar la veracidad de cada proposicin y la relacin terica que las une. Para responder este tipo de preguntas el estudiante debe leer toda la pregunta y sealar la respuesta elegida de acuerdo a las opciones presentadas. Actualmente para conectar un sensor analgico a un microcontrolador es necesario acoplar la seal con un conversor analgico digital PORQUE los microcontroladores como el PIC16F84 trabajan con 8 Bits. Seleccione una respuesta. a. la afirmacin y la razn son VERDADERAS, pero la razn NO es una explicacin CORRECTA de la afirmacin. b. la afirmacin es VERDADERA, pero la razn es una proposicin FALSA. c. la afirmacin y la razn son VERDADERAS y la razn es una explicacin CORRECTA de la afirmacin. d. la afirmacin es FALSA, pero la razn es una proposicin Correcto VERDADERA. OK muy bien. Correcto

Puntos para este envo: 1/1. Question 6 Puntos: 1 De acuerdo a la lectura, la decisin de seleccionar un sensor analgico o un sensor digital depende de: Seleccione una respuesta. a. b. c. Las capacidades del diseador El robot que necesita los sensores La cantidad de valores requeridos para medir Correcto, muy bien.

d. Las propiedades del controlador OK Correcto Puntos para este envo: 1/1.

Act 3: Reconocimiento unidad 1 Revisin del intento 1


Comenzado el Completado el viernes, 21 de marzo de 2014, 16:33 viernes, 21 de marzo de 2014, 17:13

Tiempo empleado 40 minutos 39 segundos Puntos Calificacin Question 1 Puntos: 1 Este tipo de preguntas consta de dos proposiciones, una Afirmacin y una Razn, unidas por la palabra PORQUE. Usted debe examinar la veracidad de cada proposicin y la relacin terica que las une. Para responder este tipo de preguntas usted debe leer toda la pregunta y sealar la respuesta elegida de acuerdo a las opciones presentadas. La definicin del robot industrial, como una mquina que puede efectuar un nmero diverso de trabajos, automticamente, mediante la programacin previa, no es vlida PORQUE hay robots insdustriales que no permiten programacin previa. Seleccione una respuesta. 7/7 10 de un mximo de 10 (100%)

a.

La afirmacin y la razn son VERDADERAS y la razn es

una explicacin CORRECTA de la afirmacin. b. La afirmacin es FALSA, pero la razn es una

proposicin VERDADERA. c. La afirmacin es VERDADERA, pero la razn es una Correcto, muy bien, la razn no se menciona en la lectura.

proposicin FALSA. d. La afirmacin y la razn son VERDADERAS, pero la

razn NO es una explicacin CORRECTA de la afirmacin. Correcto Puntos para este envo: 1/1. Question 2 Puntos: 1 Seleccione la opcin adecuada para completar la siguiente frase: Existen ____________ fases en el desarrollo de la robtica industrial. Seleccione una respuesta. a. b. c. d. Correcto Puntos para este envo: 1/1. Question 3 Puntos: 1 De acuerdo a la lectura, seleccione cul de las siguientes afirmaciones es verdadera. Seleccione una respuesta. Tres. Cuatro Ocho Cinco Correcto, esto se menciona textualemte en la lectura.

a.

Los precios relativos de los robots estn subiendo, y su

rentabilidad esta bajando. b. Los precios relativos de los robots estn cayendo, al igual

que su rentabilidad c. Los precios relativos de los robots estn subiendo y su

rentabilidad aumenta d. Los precios relativos de los robots estn cayendo y su Correcto, esta afirmacin es textual.

rentabilidad aumenta Correcto Puntos para este envo: 1/1. Question 4 Puntos: 1

Este tipo de preguntas consta de dos proposiciones, una Afirmacin y una Razn, unidas por la palabra PORQUE. Usted debe examinar la veracidad de cada proposicin y la relacin terica que las une. Para responder este tipo de preguntas usted debe leer toda la pregunta y sealar la respuesta elegida de acuerdo a las opciones presentadas. Los robots de servicios son idealmente adecuados al trabajo en reas demasiado peligrosas para la vida humana PORQUE han probado ser poco convenientes en situaciones de alto riesgo como en la desactivacin de bombas y en entornos contaminados radioactiva y qumicamente. Seleccione una respuesta. a. b. c. La afirmacin es VERDADERA, pero la razn es una proposicin FALSA. La afirmacin es FALSA, pero la razn es una proposicin VERDADERA. La afirmacin y la razn son VERDADERAS, pero la razn NO es una

Correcto

explicacin CORRECTA de la afirmacin. d. La afirmacin y la razn son VERDADERAS y la razn es una explicacin

CORRECTA de la afirmacin. Correcto

Puntos para este envo: 1/1. Question 5 Puntos: 1 De acuerdo a la lectura el orgen de los primeros robots industriales fu en el ao: Seleccione una respuesta. a. b. c. d. Correcto Puntos para este envo: 1/1. Question 6 Puntos: 1 De acuerdo a la lectura, la proyeccin de crecimiento estimada para los robots de correo entre 1998 hasta 2002 fu: Seleccione una respuesta. a. 15% 1948 1938 1828 1918 Muy bien.

b.

20%

c.

100%

Correcto, se espera que la cantidad de robots mencionados sea el doble, pasa de 100 a 200.

d.

50%

Correcto

Puntos para este envo: 1/1. Question 7 Puntos: 1 De las siguientes opciones seleccione cules de ellas se consideran aplicaciones de los robots de servicio: Seleccione al menos una respuesta. a. b. c. d. Correcto Puntos para este envo: 1/1. educacin ocio bienestar militar muy bien! bien bien.

omenzado el sbado, 29 de marzo de 2014, 21:46 Completado el sbado, 29 de marzo de 2014, 22:23 Tiempo empleado 37 minutos 12 segundos 7/10 Puntos 17.5 de un mximo de 25 (70%) Calificacin Question 1 Puntos: 1 De acuerdo a la nomenclatura estudiada, un robot TLR:TR cuenta con: Seleccione una respuesta. a. b. c. Cinco grados de libertad Dos grados de libertad Tres grados de libertad Excelente, muy bien.

d. Seis grados de libertad Correcto Puntos para este envo: 1/1. Question 2 Puntos: 1

De acuerodo a los tipos de espacio de trabajo mencionados, existen los siguientes: Seleccione al menos una respuesta. a. b. c. Envoltura de operacin Envoltura media Envoltura mxima Correcto, muy bien. Correcto, muy bien.

d. Envoltura restringida Correcto, muy bien. Correcto Puntos para este envo: 1/1. Question 3 Puntos: 1 Este tipo de preguntas consta de dos proposiciones, una Afirmacin y una Razn, unidas por la palabra PORQUE. usted debe examinar la veracidad de cada proposicin y la relacin terica que las une. Para responder este tipo de preguntas usted debe leer toda la pregunta y sealar la respuesta elegida de acuerdo con las opciones presentadas. El control inteligente es uno de los posibles sistemas de mando para un robot porque este sistema permite tomar decisiones basadas en el estado de los actuadores. Seleccione una respuesta. Correcto, la proposicin es a. la afirmacin es VERDADERA, pero la verdadera pero la definicin que se razn es una proposicin FALSA. OK da en la razn es falsa. b. la afirmacin y la razn son VERDADERAS, pero la razn NO es una explicacin CORRECTA de la afirmacin. c. la afirmacin es FALSA, pero la razn es una proposicin VERDADERA. d. la afirmacin y la razn son VERDADERAS y la razn es una explicacin CORRECTA de la afirmacin. Correcto Puntos para este envo: 1/1. Question 4 Puntos: 1 Teniendo en cuenta la configuracin mecnica de la mueca del robot (hoja 11) , el movimiento llamado "Pitch" corresponde con una articulacin de tipo: Seleccione una respuesta. a. Ortogonal (Traslacin) b. Rotacin (rotacion) OK c. Rotacin Incorrecto, como se observa en la notacin de la diapositiva, las

(Torsin)

primeras dos articulaciones de la mueca son tipo (R) y la ltima es (T)

d. Rotacin (Giro) Incorrecto Puntos para este envo: 0/1. Question 5 Puntos: 1 De acuerdo a la lectura seleccione cul de las siguientes afirmaciones es falsa. Seleccione una respuesta. a. La configuracin de coordenadas polares tiene 3 articulaciones b. Hay exactamente dos tipos de articulaciones de traslacin c. Hay exactamente cuatro tipos de Correcto, muy bien. son solo articulaciones de rotacin OK tres tipos (R,T,V) d. Toda articulacion tiene un eslabon de entrada y uno de salida Correcto Puntos para este envo: 1/1. Question 6 Puntos: 1 (Algo de ingles) De acuerdo al esquema de un dispositivo para programacin (pagina 19) se puede concluir que: Seleccione al menos una respuesta. a. b. c. Tal dispositivo no permite mover articulaciones individualmente. Tal dispositivo no permite grabar posiciones Tal dispositivo permite cerrar las pinzas OK Muy bien.

d. Tal dispositivo permite abrir las pinzas OK Muy bien. Correcto Puntos para este envo: 1/1. Question 7 Puntos: 1 En cuanto al espacio de trabajo seleccione cul de las siguientes informaciones es verdadera. Seleccione una respuesta. a. La envoltura mxima es mas pequea que la de operacin b. La envoltura mas pequea es la restringida

c. La envoltura restringida es mas grande que la de operacin Correcto, muy bien. OK

d.

La envoltura de operacin es mas grande que la restringida

Correcto Puntos para este envo: 1/1. Question 8 Puntos: 1 Las configuraciones que cuentan con al menos tres grados de libertad son: Seleccione al menos dos respuestas. Seleccione al menos una respuesta. a. b. c. Coordenadas polares SCARA Cilndrica Muy bien. Muy bien. Muy bien.

d. Brazo articulado Muy bien. Correcto Puntos para este envo: 1/1. Question 9 Puntos: 1 Este tipo de preguntas consta de dos proposiciones, una Afirmacin y una Razn, unidas por la palabra PORQUE. usted debe examinar la veracidad de cada proposicin y la relacin terica que las une. Para responder este tipo de preguntas usted debe leer toda la pregunta y sealar la respuesta elegida de acuerdo con las opciones presentadas. El esquema de la pgina 11 muestra que la mueca tiene tres grados de libertad porque a los movimientos generados en la mueca se les conoce como el efector final. Seleccione una respuesta. Incorrecto, la proposicin es a. la afirmacin es FALSA, pero la razn es verdadera pero la definicin que se una proposicin VERDADERA. da en la razn es falsa. b. la afirmacin y la razn son VERDADERAS, pero la razn NO es una explicacin CORRECTA de la afirmacin. c. la afirmacin es VERDADERA, pero la razn es una proposicin FALSA. OK d. la afirmacin y la razn son VERDADERAS y la razn es una explicacin CORRECTA de la afirmacin. Incorrecto Puntos para este envo: 0/1. Question 10 Puntos: 1

Este tipo de preguntas consta de dos proposiciones, una Afirmacin y una Razn, unidas por la palabra PORQUE. usted debe examinar la veracidad de cada proposicin y la relacin terica que las une. Para responder este tipo de preguntas usted debe leer toda la pregunta y sealar la respuesta elegida de acuerdo con las opciones presentadas. El esquema de la pgina 11 permite deducir que la notacin adecuada para la mueca es RRT porque los grados de libertad de este tipo de elementos en general es dos o tres. Seleccione una respuesta. a. Si la afirmacin es VERDADERA, pero la razn es una proposicin FALSA. b. Si la afirmacin y la razn son Incorrecto, muy bien las VERDADERAS y la razn es una explicacin proposiciones son verdaderas pero la razn no explica la afirmacin CORRECTA de la afirmacin. c. Si la afirmacin y la razn son VERDADERAS, pero la razn NO es una explicacin CORRECTA de la afirmacin. OK d. Si la afirmacin es FALSA, pero la razn es una proposicin VERDADERA. Incorrecto Puntos para este envo: 0/1.

Act 5: Quiz 1 Revisin del intento 1


Comenzado el Completado el sbado, 29 de marzo de 2014, 22:27 sbado, 29 de marzo de 2014, 23:25

Tiempo empleado 57 minutos 44 segundos Puntos Calificacin Question 1 Puntos: 1 9/15 21 de un mximo de 35 (60%)

Los robots industriales tienen caractersticas propias que se deben tener en cuenta para crear protocolos de seguridad adecuados en instalaciones robotizadas Cul de las siguientes caractersticas no se debe tener en cuenta para crear tales protocolos? Seleccione una respuesta. a. Campo de accin fuera del volumen cubierto por Incorrecto, esta es una caracterstica importante.

la propia mquina b. c. d. ejes Revisar la pagina 28 de mdulo Incorrecto Puntos para este envo: 0/1. Question 2 Puntos: 1 Los robots manipuladores son un subconjunto de los Seleccione una respuesta. a. b. c. d. Robots inteligentes Robots industriales Robots computador Robots de aprendizaje Correcto, muy bien. Nmero de articulaciones variable OK Movimiento simultaneo de varios ejes Movimiento independiente de cada uno d elos

Los robots manipuladores son un subconjunto de los robots industriales Correcto Puntos para este envo: 1/1.

Question 3 Puntos: 1 La Robtica contribuir en gran medida al incremento del empleo. Pero, como se puede hacer esto? al automatizar los procesos en mquinas ms flexibles, reduce el costo de maquinaria, y se produce una variedad de productos sin necesidad de realizar cambios importantes en la forma de fabricacin de los mismos. Del apartado anterior, se puede concluir correctamente que: Seleccione una respuesta. a. No es conveniente fabricar variedad de

productos con robots b. c. Un robot no es una mquina ms flexible Las mquinas automticas de la industria actual Aunque en ocasiones esto es cierto , no se puede concluir del apartado.

no permiten fabricar una variedad de productos d. Algunas mquinas automticas no son tan flexibles como los robots OK Incorrecto Puntos para este envo: 0/1. Question 4 Puntos: 1

De acuerdo al mdulo un sinnimo de articulacin es: __________________ Seleccione una respuesta. a. b. c. d. Eslabon Juntura Eslabon movil Base movil Correcto, muy bien.

Correcto Puntos para este envo: 1/1. Question 5 Puntos: 1 Este tipo de preguntas consta de dos proposiciones, una Afirmacin y una Razn, unidas por la palabra PORQUE. El estudiante debe examinar la veracidad de cada proposicin y la relacin terica que las une. Para responder este tipo de preguntas el estudiante debe leer toda la pregunta y sealar la respuesta elegida de acuerdo a las opciones presentadas: La zona de trabajo est relacionada con la cantidad de grados de libertad y a las dimensiones de los eslabones PORQUE los lmites de giro tambin restringen la zona o espacio de trabajo. Seleccione una respuesta. a. b. c. la afirmacin es VERDADERA, pero la razn es una proposicin FALSA. la afirmacin es FALSA, pero la razn es una proposicin VERDADERA. la afirmacin y la razn son VERDADERAS, pero la razn NO es una Correcto.

explicacin CORRECTA de la afirmacin. d. la afirmacin y la razn son VERDADERAS y la razn es una explicacin

CORRECTA de la afirmacin. Correcto Puntos para este envo: 1/1. Question 6 Puntos: 1 Este tipo de preguntas consta de dos proposiciones, as: una Afirmacin y una Razn, unidas por las palabra PORQUE. Usted debe examinar la veracidad de cada proposicin y la relacin terica que las une. Para responder este tipo de preguntas, debe leer toda la pregunta y sealar en las opciones de respuesta. ENUNCIADO: En robtica los errores de calibracin implican que la posicin del efector final se encuentre ligeramente alejada de la posicin requerida PORQUE

La resolucin de un robot est relacionada con el uso de sistemas digitales.

Seleccione una respuesta. a. La afirmacin y la razn son VERDADERAS y la razn es una explicacin CORRECTA de la afirmacin. b. La afirmacin y la razn son VERDADERAS pero la razn NO es una Muy bien!

explicacin CORRECTA de la afirmacin. c. d. Correcto Puntos para este envo: 1/1. Question 7 Puntos: 1 La afirmacin es VERDADERA y la razn es una proposicin falsa La razn es VERDADERA y la afirmacin es una proposicin falsa

En el contexto de la seguridad en instalaciones robotizadas, una parada de emergencia, y un pulsador de seguridad hacen parte de: _____________________________________ Seleccione una respuesta. a. b. c. d. Correcto Puntos para este envo: 1/1. Question 8 Puntos: 1 Las medidas para limitar la velocidad del robot Las caractersticas de los sistemas de control Las medidas de seguridad internas de un robot Los detectores de sobreesfuerzo de los robots Muy bien.

Seleccione de las siguientes opciones cual NO es una ventaja de la robtica en los procesos de laboratorio. Seleccione una respuesta. a. b. c. d. Aumenta la capacidad de carga OK Incrementa la productividad reduce la exposicin del humano a sustancias qumicas Mejora la calidad incorrecto

La capacidad de carga no es un parmetro importante en los procesos de laboratorio, puiesto que en general se manejan cargas livianas.. Incorrecto Puntos para este envo: 0/1. Question 9 Puntos: 1 Las consideraciones sobre la seguridad de un sistema robotizado cobran especial importancia fundamentalmente por dos razones. En primer lugar, por el motivo intrnseco de que el robot, posee mayor ndice de riesgo a un accidente que otra maquina de caractersticas similares. En segundo lugar, por un aspecto de aceptacin social del robot dentro de la fabrica, aceptacin aun dificil por lo general hoy en da. En cuanto a la seguridad en los sistemas robotizados es cierto afirmar que: Seleccione una respuesta. a. En los robots el movimiento es independiente de cada Correcto, los robots tienen uno de los ejes. Por el contrario en las mquinas independencia de movimiento convencionaleslos los movimientos son limitados y simples. de sus articulaciones. b. En los robots el campo de accin se solapa con otras mquinas, y lo mismo sucede con las mquinas convencionales.

c. En los robots existen movimientos rotacionales de los ejes. Por el contrario en las mquinas convencionales los movimientos son lineales. d. En los robots la programacin de movimientos es ms compleja. Por el contrario las mquinas convencionales en ocasiones ni siquiera requieren programacin Correcto Puntos para este envo: 1/1. Question 10 Puntos: 1 Este tipo de pregunta consta de un enunciado, problema o contexto, frente al cual, usted debe seleccionar la opcin de respuesta que responda correctamente a la pregunta: ENUNCIADO Teniendo en cuenta la definicin de grados de libertad, podemos afirmar que un ROBOT que est compuesto por una articulacin prismtica y una articulacin esfrica cuenta con: Seleccione una respuesta. a. tres grados de Incorrecto, una articulacin esfrica y una prismtica tienen por si solas 3 y un grado de libertad respectivamente.

libertad b. Cinco grados de

libertad c. Dos grados de

libertad d. Cuatro grados de libertad OK Incorrecto Puntos para este envo: 0/1. Question 11 Puntos: 1

Contexto: Este tipo de pregunta se desarrolla en torno a un (1) enunciado y cuatro (4) opciones de respuesta. Solo una (1) de estas opciones responde correctamente a la pregunta. Enunciado: Se suele pensar que mientras ms rasgos humanoides se incorporen al robot, mayor familiaridad despertar, y ms fluida y agradable ser la interaccin del hombre con la mquina. Por este motivo se tiende a dotar a algunos robots que van a interactuar habitualmente con humanos (por ejemplo, los de servicio o los de exhibicin) de elementos totalmente superfluos desde el punto de vista funcional, como nariz o cejas. En el anterior aparte del mdulo, la palabra superfluos se puede cambiar por ____________________ sin cambiar el sentido de la lectura. Seleccione una respuesta. a. Algunas veces necesarios

b.

Necesarios Correcto, , los elementos mencionados son totalmente innecesarios

c.

Innecesarios

d. Correcto

Insuficientes.

Puntos para este envo: 1/1. Question 12 Puntos: 1 Contexto: Este tipo de pregunta se desarrolla en torno a un (1) enunciado y cuatro (4) opciones de respuesta. Solo una (1) de estas opciones responde correctamente a la pregunta Enunciado: Debido al constante cambio tecnolgico que vivimos en esta poca de la historia, no existe un acuerdo sobre la definicin formal de robot, por ejemplo el Diccionario Merrian Webster dice que un robot es: Una mquina que se asemeja a los humanos y desarrolla como ellos tareas complejas como andar o hablar. Desde el punto de vista del diccionario mencionado una de las siguientes conclusiones es vlida: Seleccione una respuesta.

a. b. c. d.

Los carros seguidores de linea hacen parte de esta definicin. Los robots de servicio hacen parte de esta definicin. OK Los sistemas de audio son parte de esta definicin. Los controles de temperatura son sistemas robticos Incorrecto.

Incorrecto Puntos para este envo: 0/1. Question 13 Puntos: 1 El subsistema que alimenta los actuadores en un robot es llamado: Seleccione una respuesta. a. b. c. d. Correcto Puntos para este envo: 1/1. Question 14 Puntos: 1 Este tipo de preguntas consta de dos proposiciones, una Afirmacin y una Razn, unidas por la palabra PORQUE. El estudiante debe examinar la veracidad de cada proposicin y la relacin terica que las une. Para responder este tipo de preguntas el estudiante debe leer toda la pregunta y sealar la respuesta elegida de acuerdo a las opciones presentadas: Una causa para que los robots no puedan llegar a todas las posiciones dentro del espacio de trabajo es la resolucin finita provocada por los sistemas digitales PORQUE en esto caso los usuarios deben asegurarse que el robot llegue a la posicin discreta ms cercana. Subsistema Elctrico Subsistema de conexin Subsistema de potencia Subsistema alimentador Muy bien.

Seleccione una respuesta. a. la afirmacin y la razn son VERDADERAS, pero la razn NO es una explicacin CORRECTA de la afirmacin. OK b. la afirmacin es FALSA, pero la razn es una proposicin VERDADERA. c. la afirmacin es VERDADERA, pero la razn es una proposicin FALSA. d. la afirmacin y la razn son VERDADERAS y la razn es una explicacin CORRECTA de la afirmacin. Incorrecto Puntos para este envo: 0/1. Question 15 Puntos: 1 La zona de trabajo de un robot manipulador se ve restringida por: Seleccione al menos una respuesta Seleccione al menos una respuesta. a. b. c. d. los limites de giro de sus articulaciones los limites de desplazamiento de sus articulaciones los lmites elctricos de los actuadores la cantidad de articulaciones lo que es cierto es que la necesidad de ubicar el Incorrecto. robot en un punto cercano surge del problema de resolucin finita, y no que el ubicar el robot en punto cercano provoque la resolucin finita.

La cantidad de articulaciones est directamente relacionada con los grados de libertad, y por tanto con la zona de trabajo.

Correcto Puntos para este envo: 1/1.

Act 7 : Reconocimiento Unidad 2


Question 1 Puntos: 1 De acuerdo a las definiciones presentadas, determine cul de las siguientes afirmaciones es verdadera. Seleccione una respuesta. a. b. c. d. Es necesario que una matriz sea simtrica para que sea cuadrada. Es necesario que una matriz sea cuadrada para que sea simtrica . Es necesario que una matriz sea cuadrada para que sea triangular superior. Es necesario que una matriz sea triangular superior para que sea cuadrada.

Question 2 Puntos: 1 De acuerdo a las definiciones presentadas, determine cul de las siguientes afirmaciones es verdadera. Seleccione una respuesta. a. b. c. d. Una matriz de mxn siempre debe ser cuadarada Una matriz de mxn tiene "m" columnas y "n" filas. Una matriz de nxm tiene "m" filas y "n" columnas. Una matriz de mxn tiene "m" filas y "n" columnas

Question 3 Puntos: 1

Teniendo en cuenta el ejemplo 3 para el tema de vectores en el plano, hllese el vector cuyo punto inicial es (1,1) y cuyo punto final es (1,2). Seleccione una respuesta. a. b. c. d. (1,-1) (0,1) (2,0) (0,-1)

Question 4 Puntos: 1 De acuerdo a las definiciones del libro, una matriz B de mxn tiene: Seleccione al menos una respuesta. a. b. c. d. n columnas m filas n filas m columnas

Question 5 Puntos: 1 Dados dos vectores en el espacio tridimensional v1=(1,-2,0) y v2=(2,-2,1) el resultado de sumarlos genera: Seleccione al menos una respuesta. a. b. (3,4,1) (3,-4,1)

c. d.

Un vector tridimensional Un vector bidimensional

Question 6 Puntos: 1 Dados los vectores en el espacio tridimensional U=(0,0,1) V=(0,1,0) W=(1,0,1), y con T=U+V+W, determine Cul es el ngulo de T? Seleccione una respuesta. a. b. c. d. Falta un dato en la pregunta. El ngulo es 45 grados El ngulo es 90 grados El ngulo es 60 grados

Vous aimerez peut-être aussi